TABE Reading Practice Test: Level A

This TABE Reading practice test covers Level A. This is the advanced level, so the questions are extra challenging. You may notice that these passages are more complex with numerous details. The questions will test your reading comprehension skills and your ability to infer implied details.

Congratulations - you have completed . You scored %%SCORE%% out of %%TOTAL%%. Your performance has been rated as %%RATING%%
Your answers are highlighted below.
Question 1
The Spark that Lit the Fire

(1) On March 3, 1991, Rodney King, a construction worker from Los Angeles, was driving along California State Route 210 with two friends when police moved to pull him over. King, by his own admission, was on the way home from drinking at a friend’s house and had been drinking. In an attempt to avoid the consequences of driving under the influence, King led the police on a high-speed chase. When King chose to leave the freeway and continue to flee from the police on residential streets, several more police cars and even a police helicopter joined the chase. After about 8 miles of chasing King, the police were successfully able to corner his car and force King to surrender.

(2) It is at this moment that the controversy began. King’s two friends, both African American males, got out of the car, and by their own accounts, were mistreated by police officers. Bryant Allen claims he was beaten and stomped as he lay on the ground; Freddie Helms ended up with a cut on his head. King, an African American man himself, was reluctant to get out of the car, and when he finally did, LAPD officers swarmed him and used force to try to subdue him. A man in a nearby apartment heard the commotion and began filming the incident on a camcorder. The video footage begins with King already on the ground, and shows LAPD officers stomping King, beating him with batons, and generally using an amount of force unnecessary to subdue a man who was already on the ground.

(3) King went to the hospital, and four of the officers involved in the incident were charged with assault and use of excessive force. Despite the video clearly showing these officers beating King, the four officers were essentially acquitted of their charges. Jurors acquitted all four officers on assault charges and while they only acquitted three of the officers on charges of excessive force, they were unable to reach a decision on the fourth.

(4) The Rodney King incident was not an isolated incident; it became a very public display of a problem that African Americans had been struggling with for a long time: the police’s mistreatment of people of color. When word reached the streets of Los Angeles that these four officers had been acquitted, the community exploded. This verdict sparked the infamous LA riots, which lasted 6 days, caused 63 deaths, damaged thousands of businesses and reportedly caused almost $1 billion in financial losses. The people of Los Angeles had had enough of police abusing them and getting away with it. Police brutality is still a problem today, and all because Rodney King’s abusers went free.
 

Read the sentence from Paragraph 2.

It is at this moment that the controversy began.

Which key idea does this sentence support?

A
King and his friends should not have been drinking and driving.
B
Police officers were justified in the amount of force they used to subdue Rodney King.
C
The events of this arrest will lead to a much bigger problem.
D
Police officers had already shown that they were not concerned about harming King and his friends.
Question 1 Explanation: 
This sentence marks a turning point in the passage. Up until now, the story was a straightforward description of a police chase. King and his friends should not have been speeding and driving drunk, and the police were justified in trying to pull them over and subsequently arrest them. The narrative changes from this moment forward to one of abusive police officers using excessive force. While answer choice (A) might be true, it is not supported by this sentence, and answer choices (B) and (D) are note supported by the passage.
Question 2
The Spark that Lit the Fire

(1) On March 3, 1991, Rodney King, a construction worker from Los Angeles, was driving along California State Route 210 with two friends when police moved to pull him over. King, by his own admission, was on the way home from drinking at a friend’s house and had been drinking. In an attempt to avoid the consequences of driving under the influence, King led the police on a high-speed chase. When King chose to leave the freeway and continue to flee from the police on residential streets, several more police cars and even a police helicopter joined the chase. After about 8 miles of chasing King, the police were successfully able to corner his car and force King to surrender.

(2) It is at this moment that the controversy began. King’s two friends, both African American males, got out of the car, and by their own accounts, were mistreated by police officers. Bryant Allen claims he was beaten and stomped as he lay on the ground; Freddie Helms ended up with a cut on his head. King, an African American man himself, was reluctant to get out of the car, and when he finally did, LAPD officers swarmed him and used force to try to subdue him. A man in a nearby apartment heard the commotion and began filming the incident on a camcorder. The video footage begins with King already on the ground, and shows LAPD officers stomping King, beating him with batons, and generally using an amount of force unnecessary to subdue a man who was already on the ground.

(3) King went to the hospital, and four of the officers involved in the incident were charged with assault and use of excessive force. Despite the video clearly showing these officers beating King, the four officers were essentially acquitted of their charges. Jurors acquitted all four officers on assault charges and while they only acquitted three of the officers on charges of excessive force, they were unable to reach a decision on the fourth.

(4) The Rodney King incident was not an isolated incident; it became a very public display of a problem that African Americans had been struggling with for a long time: the police’s mistreatment of people of color. When word reached the streets of Los Angeles that these four officers had been acquitted, the community exploded. This verdict sparked the infamous LA riots, which lasted 6 days, caused 63 deaths, damaged thousands of businesses and reportedly caused almost $1 billion in financial losses. The people of Los Angeles had had enough of police abusing them and getting away with it. Police brutality is still a problem today, and all because Rodney King’s abusers went free.
 

Which of these is most likely the author’s purpose for writing the article?

A
The author wants to inspire the reader to fight for more oversight within urban police departments.
B
The author wants to provide a unique perspective on a well-known story in order to try to pen the readers’ minds.
C
The author wants to show the readers that justice may come in unexpected ways, but it is always served.
D
The author wants the reader to know how the Rodney King incident directly caused the LA riots of 1992.
Question 2 Explanation: 
While the author does insert their opinion into the article, the main purpose is to show the causal relationship between the Rodney King beating and the LA riots. Answer choice (A) might be tempting because the author would likely support more oversight, but the article doesn’t discuss that at all. The author’s perspective is not “unique,” as suggested in answer choice (B), as it is a fairly widely held opinion that the officers used excessive force on Rodney King and that this incident was a catalyst for the LA riots. Answer choice (C) is also incorrect because the author doesn’t leave the reader with a sense that justice has been served.
Question 3
The Spark that Lit the Fire

(1) On March 3, 1991, Rodney King, a construction worker from Los Angeles, was driving along California State Route 210 with two friends when police moved to pull him over. King, by his own admission, was on the way home from drinking at a friend’s house and had been drinking. In an attempt to avoid the consequences of driving under the influence, King led the police on a high-speed chase. When King chose to leave the freeway and continue to flee from the police on residential streets, several more police cars and even a police helicopter joined the chase. After about 8 miles of chasing King, the police were successfully able to corner his car and force King to surrender.

(2) It is at this moment that the controversy began. King’s two friends, both African American males, got out of the car, and by their own accounts, were mistreated by police officers. Bryant Allen claims he was beaten and stomped as he lay on the ground; Freddie Helms ended up with a cut on his head. King, an African American man himself, was reluctant to get out of the car, and when he finally did, LAPD officers swarmed him and used force to try to subdue him. A man in a nearby apartment heard the commotion and began filming the incident on a camcorder. The video footage begins with King already on the ground, and shows LAPD officers stomping King, beating him with batons, and generally using an amount of force unnecessary to subdue a man who was already on the ground.

(3) King went to the hospital, and four of the officers involved in the incident were charged with assault and use of excessive force. Despite the video clearly showing these officers beating King, the four officers were essentially acquitted of their charges. Jurors acquitted all four officers on assault charges and while they only acquitted three of the officers on charges of excessive force, they were unable to reach a decision on the fourth.

(4) The Rodney King incident was not an isolated incident; it became a very public display of a problem that African Americans had been struggling with for a long time: the police’s mistreatment of people of color. When word reached the streets of Los Angeles that these four officers had been acquitted, the community exploded. This verdict sparked the infamous LA riots, which lasted 6 days, caused 63 deaths, damaged thousands of businesses and reportedly caused almost $1 billion in financial losses. The people of Los Angeles had had enough of police abusing them and getting away with it. Police brutality is still a problem today, and all because Rodney King’s abusers went free.
 

Part A

How does the author provide effective support for the main idea of the article?

A
The author suggests that this incident was the most public example of a systemic problem.
B
The author demonizes Rodney King in order to justify the actions of the police officers.
C
The author provides incriminating background information on the arresting officers.
D
The author sets Rodney King up as a victim of police harassment from the very beginning.
Question 3 Explanation: 
In an attempt to provide context for the LA riots, the author points out that this was not the only incident of police brutality towards people of color, but rather the most public.
Question 4
The Spark that Lit the Fire

(1) On March 3, 1991, Rodney King, a construction worker from Los Angeles, was driving along California State Route 210 with two friends when police moved to pull him over. King, by his own admission, was on the way home from drinking at a friend’s house and had been drinking. In an attempt to avoid the consequences of driving under the influence, King led the police on a high-speed chase. When King chose to leave the freeway and continue to flee from the police on residential streets, several more police cars and even a police helicopter joined the chase. After about 8 miles of chasing King, the police were successfully able to corner his car and force King to surrender.

(2) It is at this moment that the controversy began. King’s two friends, both African American males, got out of the car, and by their own accounts, were mistreated by police officers. Bryant Allen claims he was beaten and stomped as he lay on the ground; Freddie Helms ended up with a cut on his head. King, an African American man himself, was reluctant to get out of the car, and when he finally did, LAPD officers swarmed him and used force to try to subdue him. A man in a nearby apartment heard the commotion and began filming the incident on a camcorder. The video footage begins with King already on the ground, and shows LAPD officers stomping King, beating him with batons, and generally using an amount of force unnecessary to subdue a man who was already on the ground.

(3) King went to the hospital, and four of the officers involved in the incident were charged with assault and use of excessive force. Despite the video clearly showing these officers beating King, the four officers were essentially acquitted of their charges. Jurors acquitted all four officers on assault charges and while they only acquitted three of the officers on charges of excessive force, they were unable to reach a decision on the fourth.

(4) The Rodney King incident was not an isolated incident; it became a very public display of a problem that African Americans had been struggling with for a long time: the police’s mistreatment of people of color. When word reached the streets of Los Angeles that these four officers had been acquitted, the community exploded. This verdict sparked the infamous LA riots, which lasted 6 days, caused 63 deaths, damaged thousands of businesses and reportedly caused almost $1 billion in financial losses. The people of Los Angeles had had enough of police abusing them and getting away with it. Police brutality is still a problem today, and all because Rodney King’s abusers went free.
 

Part B

Which detail from the article best supports the answer to Part A?

A
“The Rodney King incident was not an isolated incident; it became a very public display of a problem that African Americans had been struggling with for a long time: the police’s mistreatment of people of color.”
B
“King, by his own admission, was on the way home from drinking at a friend’s house and had been drinking.”
C
“King went to the hospital, and four of the officers involved in the incident were charged with assault and use of excessive force.”
D
“After about 8 miles of chasing King, the police were successfully able to corner his car and force King to surrender.”
Question 4 Explanation: 
Answer A most clearly supports the author's main idea. Even though the incident was caught on camera and gained national attention, officers were still not punished. People felt angry and hopeless. The author is quick to show that King had been breaking the law, but he is not demonizing him. Even if you have broken the law, you have rights that protect you from excessive force. There is also nothing in the article to suggest that police had done anything wrong until King’s car was pulled over.
Question 5
The Spark that Lit the Fire

(1) On March 3, 1991, Rodney King, a construction worker from Los Angeles, was driving along California State Route 210 with two friends when police moved to pull him over. King, by his own admission, was on the way home from drinking at a friend’s house and had been drinking. In an attempt to avoid the consequences of driving under the influence, King led the police on a high-speed chase. When King chose to leave the freeway and continue to flee from the police on residential streets, several more police cars and even a police helicopter joined the chase. After about 8 miles of chasing King, the police were successfully able to corner his car and force King to surrender.

(2) It is at this moment that the controversy began. King’s two friends, both African American males, got out of the car, and by their own accounts, were mistreated by police officers. Bryant Allen claims he was beaten and stomped as he lay on the ground; Freddie Helms ended up with a cut on his head. King, an African American man himself, was reluctant to get out of the car, and when he finally did, LAPD officers swarmed him and used force to try to subdue him. A man in a nearby apartment heard the commotion and began filming the incident on a camcorder. The video footage begins with King already on the ground, and shows LAPD officers stomping King, beating him with batons, and generally using an amount of force unnecessary to subdue a man who was already on the ground.

(3) King went to the hospital, and four of the officers involved in the incident were charged with assault and use of excessive force. Despite the video clearly showing these officers beating King, the four officers were essentially acquitted of their charges. Jurors acquitted all four officers on assault charges and while they only acquitted three of the officers on charges of excessive force, they were unable to reach a decision on the fourth.

(4) The Rodney King incident was not an isolated incident; it became a very public display of a problem that African Americans had been struggling with for a long time: the police’s mistreatment of people of color. When word reached the streets of Los Angeles that these four officers had been acquitted, the community exploded. This verdict sparked the infamous LA riots, which lasted 6 days, caused 63 deaths, damaged thousands of businesses and reportedly caused almost $1 billion in financial losses. The people of Los Angeles had had enough of police abusing them and getting away with it. Police brutality is still a problem today, and all because Rodney King’s abusers went free.
 

Part A

Which statement explains how paragraphs 3 and 4 develop the author’s claim that police officers acted improperly during their arrest of Rodney King?

A
Paragraph 3 describes the police officer’s assault in detail, and Paragraph 4 details the missteps during their trial.
B
Paragraph 3 juxtaposes Rodney King’s minimal crimes against the brutal acts of police officers portrayed in Paragraph 4.
C
Paragraph 3 shows the indecisiveness of the juror during the officers’ trial, and Paragraph 4 condemns the jury for not rendering a better decision.
D
Paragraph 3 establishes the police officers as guilty of assault and use of excessive force, and Paragraph 4 presents the ramifications of their acquittal.
Question 5 Explanation: 
Only answer choice (D) accurately reflects Paragraph 3 and 4. Paragraph 3 expresses the verdict as unbelievable considering the circumstances, and Paragraph 4 discusses the aftermath.
Question 6
The Spark that Lit the Fire

(1) On March 3, 1991, Rodney King, a construction worker from Los Angeles, was driving along California State Route 210 with two friends when police moved to pull him over. King, by his own admission, was on the way home from drinking at a friend’s house and had been drinking. In an attempt to avoid the consequences of driving under the influence, King led the police on a high-speed chase. When King chose to leave the freeway and continue to flee from the police on residential streets, several more police cars and even a police helicopter joined the chase. After about 8 miles of chasing King, the police were successfully able to corner his car and force King to surrender.

(2) It is at this moment that the controversy began. King’s two friends, both African American males, got out of the car, and by their own accounts, were mistreated by police officers. Bryant Allen claims he was beaten and stomped as he lay on the ground; Freddie Helms ended up with a cut on his head. King, an African American man himself, was reluctant to get out of the car, and when he finally did, LAPD officers swarmed him and used force to try to subdue him. A man in a nearby apartment heard the commotion and began filming the incident on a camcorder. The video footage begins with King already on the ground, and shows LAPD officers stomping King, beating him with batons, and generally using an amount of force unnecessary to subdue a man who was already on the ground.

(3) King went to the hospital, and four of the officers involved in the incident were charged with assault and use of excessive force. Despite the video clearly showing these officers beating King, the four officers were essentially acquitted of their charges. Jurors acquitted all four officers on assault charges and while they only acquitted three of the officers on charges of excessive force, they were unable to reach a decision on the fourth.

(4) The Rodney King incident was not an isolated incident; it became a very public display of a problem that African Americans had been struggling with for a long time: the police’s mistreatment of people of color. When word reached the streets of Los Angeles that these four officers had been acquitted, the community exploded. This verdict sparked the infamous LA riots, which lasted 6 days, caused 63 deaths, damaged thousands of businesses and reportedly caused almost $1 billion in financial losses. The people of Los Angeles had had enough of police abusing them and getting away with it. Police brutality is still a problem today, and all because Rodney King’s abusers went free.
 

Part B

Which sentence from Paragraph 3 or 4 best supports the answer to Part A?

A
“King went to the hospital, and four of the officers involved in the incident were charged with assault and use of excessive force.”
B
“Despite the video clearly showing these officers beating King, the four officers were essentially acquitted of their charges.”
C
“Jurors acquitted all four officers on assault charges and while they only acquitted three of the officers on charges of excessive force, they were unable to reach a decision on the fourth.”
D
“The video footage begins with King already on the ground, and shows LAPD officers stomping King, beating him with batons, and generally using an amount of force unnecessary to subdue a man who was already on the ground.”
Question 6 Explanation: 
While both answer choice (B) and (D) support the ideas of Part A, answer choice (D) is from Paragraph 2, and therefore does not fit the criteria of the question.
Question 7
The Spark that Lit the Fire

(1) On March 3, 1991, Rodney King, a construction worker from Los Angeles, was driving along California State Route 210 with two friends when police moved to pull him over. King, by his own admission, was on the way home from drinking at a friend’s house and had been drinking. In an attempt to avoid the consequences of driving under the influence, King led the police on a high-speed chase. When King chose to leave the freeway and continue to flee from the police on residential streets, several more police cars and even a police helicopter joined the chase. After about 8 miles of chasing King, the police were successfully able to corner his car and force King to surrender.

(2) It is at this moment that the controversy began. King’s two friends, both African American males, got out of the car, and by their own accounts, were mistreated by police officers. Bryant Allen claims he was beaten and stomped as he lay on the ground; Freddie Helms ended up with a cut on his head. King, an African American man himself, was reluctant to get out of the car, and when he finally did, LAPD officers swarmed him and used force to try to subdue him. A man in a nearby apartment heard the commotion and began filming the incident on a camcorder. The video footage begins with King already on the ground, and shows LAPD officers stomping King, beating him with batons, and generally using an amount of force unnecessary to subdue a man who was already on the ground.

(3) King went to the hospital, and four of the officers involved in the incident were charged with assault and use of excessive force. Despite the video clearly showing these officers beating King, the four officers were essentially acquitted of their charges. Jurors acquitted all four officers on assault charges and while they only acquitted three of the officers on charges of excessive force, they were unable to reach a decision on the fourth.

(4) The Rodney King incident was not an isolated incident; it became a very public display of a problem that African Americans had been struggling with for a long time: the police’s mistreatment of people of color. When word reached the streets of Los Angeles that these four officers had been acquitted, the community exploded. This verdict sparked the infamous LA riots, which lasted 6 days, caused 63 deaths, damaged thousands of businesses and reportedly caused almost $1 billion in financial losses. The people of Los Angeles had had enough of police abusing them and getting away with it. Police brutality is still a problem today, and all because Rodney King’s abusers went free.
 

How does the author advance their point of view in the article?

A
The author presents two perspectives on a controversial issue.
B
The author takes readers through a detailed account of the incident and the aftermath.
C
The author uses data and statistics to support their assertions.
D
The author presents unbiased information and leaves it up to the readers to decide for themselves.
Question 7 Explanation: 
The author takes readers through a chronological accounting of the incident, only introducing their opinion at the end of the second paragraph. The author doesn’t present two perspectives (A), support their opinion with specific data (C), or remain unbiased throughout (D).
Question 8
The Spark that Lit the Fire

(1) On March 3, 1991, Rodney King, a construction worker from Los Angeles, was driving along California State Route 210 with two friends when police moved to pull him over. King, by his own admission, was on the way home from drinking at a friend’s house and had been drinking. In an attempt to avoid the consequences of driving under the influence, King led the police on a high-speed chase. When King chose to leave the freeway and continue to flee from the police on residential streets, several more police cars and even a police helicopter joined the chase. After about 8 miles of chasing King, the police were successfully able to corner his car and force King to surrender.

(2) It is at this moment that the controversy began. King’s two friends, both African American males, got out of the car, and by their own accounts, were mistreated by police officers. Bryant Allen claims he was beaten and stomped as he lay on the ground; Freddie Helms ended up with a cut on his head. King, an African American man himself, was reluctant to get out of the car, and when he finally did, LAPD officers swarmed him and used force to try to subdue him. A man in a nearby apartment heard the commotion and began filming the incident on a camcorder. The video footage begins with King already on the ground, and shows LAPD officers stomping King, beating him with batons, and generally using an amount of force unnecessary to subdue a man who was already on the ground.

(3) King went to the hospital, and four of the officers involved in the incident were charged with assault and use of excessive force. Despite the video clearly showing these officers beating King, the four officers were essentially acquitted of their charges. Jurors acquitted all four officers on assault charges and while they only acquitted three of the officers on charges of excessive force, they were unable to reach a decision on the fourth.

(4) The Rodney King incident was not an isolated incident; it became a very public display of a problem that African Americans had been struggling with for a long time: the police’s mistreatment of people of color. When word reached the streets of Los Angeles that these four officers had been acquitted, the community exploded. This verdict sparked the infamous LA riots, which lasted 6 days, caused 63 deaths, damaged thousands of businesses and reportedly caused almost $1 billion in financial losses. The people of Los Angeles had had enough of police abusing them and getting away with it. Police brutality is still a problem today, and all because Rodney King’s abusers went free.
 

Read this sentence from Paragraph 4:

Police brutality is still a problem today, and all because Rodney King’s abusers went free.

Has the author effectively supported this claim?

A
No; the author points out that while the police officers were initially acquitted, they each ended up spending time in jail for the incident.
B
No; the author wrote earlier in the passage that this brutality had been happening long before the Rodney King incident.
C
Yes; the LA riots sparked a counter-movement within the police that arguably resulted in more violence towards people of color.
D
Yes; the author clearly establishes that this was the first instance of police brutality going unpunished in California history.
Question 8 Explanation: 
Answer choice (B) is correct because the author directly refutes this conclusive sentence within the article. The Rodney King was the most public instance of police brutality against people of colors, but it certainly wasn’t the first, and nor would it be the last.
Question 9
The Spark that Lit the Fire

(1) On March 3, 1991, Rodney King, a construction worker from Los Angeles, was driving along California State Route 210 with two friends when police moved to pull him over. King, by his own admission, was on the way home from drinking at a friend’s house and had been drinking. In an attempt to avoid the consequences of driving under the influence, King led the police on a high-speed chase. When King chose to leave the freeway and continue to flee from the police on residential streets, several more police cars and even a police helicopter joined the chase. After about 8 miles of chasing King, the police were successfully able to corner his car and force King to surrender.

(2) It is at this moment that the controversy began. King’s two friends, both African American males, got out of the car, and by their own accounts, were mistreated by police officers. Bryant Allen claims he was beaten and stomped as he lay on the ground; Freddie Helms ended up with a cut on his head. King, an African American man himself, was reluctant to get out of the car, and when he finally did, LAPD officers swarmed him and used force to try to subdue him. A man in a nearby apartment heard the commotion and began filming the incident on a camcorder. The video footage begins with King already on the ground, and shows LAPD officers stomping King, beating him with batons, and generally using an amount of force unnecessary to subdue a man who was already on the ground.

(3) King went to the hospital, and four of the officers involved in the incident were charged with assault and use of excessive force. Despite the video clearly showing these officers beating King, the four officers were essentially acquitted of their charges. Jurors acquitted all four officers on assault charges and while they only acquitted three of the officers on charges of excessive force, they were unable to reach a decision on the fourth.

(4) The Rodney King incident was not an isolated incident; it became a very public display of a problem that African Americans had been struggling with for a long time: the police’s mistreatment of people of color. When word reached the streets of Los Angeles that these four officers had been acquitted, the community exploded. This verdict sparked the infamous LA riots, which lasted 6 days, caused 63 deaths, damaged thousands of businesses and reportedly caused almost $1 billion in financial losses. The people of Los Angeles had had enough of police abusing them and getting away with it. Police brutality is still a problem today, and all because Rodney King’s abusers went free.
 

Which of the following words from Paragraph 2 best conveys the author’s opinion?

A
subdue
B
force
C
claims
D
unnecessary
Question 9 Explanation: 
“Unnecessary” is the first word that the author uses that shows opinion. The author is laying judgment on the incident. The other answer choices are not biased in context. Answer choice (A) might be tempting, but it was literally the police officers’ job to “subdue” King. It is only when they use “unnecessary” force to do so that the problem arises.
Question 10
The Spark that Lit the Fire

(1) On March 3, 1991, Rodney King, a construction worker from Los Angeles, was driving along California State Route 210 with two friends when police moved to pull him over. King, by his own admission, was on the way home from drinking at a friend’s house and had been drinking. In an attempt to avoid the consequences of driving under the influence, King led the police on a high-speed chase. When King chose to leave the freeway and continue to flee from the police on residential streets, several more police cars and even a police helicopter joined the chase. After about 8 miles of chasing King, the police were successfully able to corner his car and force King to surrender.

(2) It is at this moment that the controversy began. King’s two friends, both African American males, got out of the car, and by their own accounts, were mistreated by police officers. Bryant Allen claims he was beaten and stomped as he lay on the ground; Freddie Helms ended up with a cut on his head. King, an African American man himself, was reluctant to get out of the car, and when he finally did, LAPD officers swarmed him and used force to try to subdue him. A man in a nearby apartment heard the commotion and began filming the incident on a camcorder. The video footage begins with King already on the ground, and shows LAPD officers stomping King, beating him with batons, and generally using an amount of force unnecessary to subdue a man who was already on the ground.

(3) King went to the hospital, and four of the officers involved in the incident were charged with assault and use of excessive force. Despite the video clearly showing these officers beating King, the four officers were essentially acquitted of their charges. Jurors acquitted all four officers on assault charges and while they only acquitted three of the officers on charges of excessive force, they were unable to reach a decision on the fourth.

(4) The Rodney King incident was not an isolated incident; it became a very public display of a problem that African Americans had been struggling with for a long time: the police’s mistreatment of people of color. When word reached the streets of Los Angeles that these four officers had been acquitted, the community exploded. This verdict sparked the infamous LA riots, which lasted 6 days, caused 63 deaths, damaged thousands of businesses and reportedly caused almost $1 billion in financial losses. The people of Los Angeles had had enough of police abusing them and getting away with it. Police brutality is still a problem today, and all because Rodney King’s abusers went free.
 

Which of the following statements best represents an inference that can be made from the text?

A
The police officers who were charged deserved the punishment they received for their actions.
B
Police officers should not be allowed to use force to subdue a suspect.
C
The police officers were justified in pulling Rodney King and his friends over.
D
Rodney King and his friends were innocent citizens who were unfairly targeted by the police.
Question 10 Explanation: 
Answer choice (C) is correct because the entire 1st paragraph describes what could have been a legal and appropriate response from police. They attempted to pull over someone who was speeding, and responded with more police units when the suspects led them on a high-speed chase that was becoming increasingly dangerous. It was only after the suspects got out of the car that they were treated inappropriately. This directly refutes answer choice (D), and while (A) might be tempting, the author points out that the officers are not punished.
Question 11
Saving the Environment

(1) Preserving the environment is a monumental task, and people often have difficulty seeing the connection between what they do and the global problem. Considering how massive the problem is, shaming someone for not recycling a bottle is like yelling at someone for adding a drop of water to a flood. That one bottle is not going to ruin the environment, but 7 billion bottles will. It is certainly important for each citizen to do their part to help improve the world, but too often the onus of environmental preservation is thrust upon the least culpable offenders. Why is our attention so often focused on the daily impact of ordinary citizens rather than the corporations who are doing infinitely more harm every day?

(2) A study called the Carbon Majors Report, conducted in 2015, identified just 100 companies as responsible for 71% of global emissions. These companies are doing far more damage to the environment than average citizens, but America continues to pass legislation that softens environmental protections, allowing companies to continue striving for profit over the health of our globe. While allowing these companies to operate free of restrictions might cause a small uptick in employment, that is a very shortsighted reward considering the consequences.

(3) Governments allow these companies to continue to poison our globe while they demonize the average citizen for their minimal contribution to the problem, or worse: deny that the problem even exists. Problems don’t go away because you ignore them, and until companies are held responsible for their destructive emissions, things are only going to get worse.

(4) Perhaps the most frustrating part of our environmental woes is that the majority of the “problem” companies are invested in industries that could easily be on their way out, like coal and crude oil. Instead of pivoting and investing in new fuels, new energy sources, etc., these companies continue to bank on a dying industry. Scientists have made a lot of progress in clean energy and power, but until companies are willing to make real adjustments, we’re stuck in the stone age pumping our atmosphere full of poisons.

(5) The only way we can force companies to strive for real environmental preservation is if we continue fighting for legislation that compels them to do so. Instead of just asking average citizens to put their water bottles in a recycling bin, we can ask them to also vote with this issue in mind. Elect the right officials who will create and adopt the right legislation that will force companies to act responsibly. The fate of the globe depends on it.
 

Which of the following sentence from Paragraph 1 hurts the author’s overall argument?

A
“Preserving the environment is a monumental task, and people often have difficulty seeing the connection between what they do and the global problem.”
B
“Considering how massive the problem is, shaming someone for not recycling a bottle is like yelling at someone for adding a drop of water to a flood.”
C
“That one bottle is not going to ruin the environment, but 7 billion bottles will.”
D
It is certainly important for each citizen to do their part to help improve the world, but too often the onus of environmental preservation is thrust upon the least culpable offenders.”
Question 11 Explanation: 
Answer choice (C) is correct because it undermines the point that the author is trying to make. The author wants to convince the audience that corporations need to change more than the average citizens do, but this sentence brings the focus back on the average citizen.
Question 12
Saving the Environment

(1) Preserving the environment is a monumental task, and people often have difficulty seeing the connection between what they do and the global problem. Considering how massive the problem is, shaming someone for not recycling a bottle is like yelling at someone for adding a drop of water to a flood. That one bottle is not going to ruin the environment, but 7 billion bottles will. It is certainly important for each citizen to do their part to help improve the world, but too often the onus of environmental preservation is thrust upon the least culpable offenders. Why is our attention so often focused on the daily impact of ordinary citizens rather than the corporations who are doing infinitely more harm every day?

(2) A study called the Carbon Majors Report, conducted in 2015, identified just 100 companies as responsible for 71% of global emissions. These companies are doing far more damage to the environment than average citizens, but America continues to pass legislation that softens environmental protections, allowing companies to continue striving for profit over the health of our globe. While allowing these companies to operate free of restrictions might cause a small uptick in employment, that is a very shortsighted reward considering the consequences.

(3) Governments allow these companies to continue to poison our globe while they demonize the average citizen for their minimal contribution to the problem, or worse: deny that the problem even exists. Problems don’t go away because you ignore them, and until companies are held responsible for their destructive emissions, things are only going to get worse.

(4) Perhaps the most frustrating part of our environmental woes is that the majority of the “problem” companies are invested in industries that could easily be on their way out, like coal and crude oil. Instead of pivoting and investing in new fuels, new energy sources, etc., these companies continue to bank on a dying industry. Scientists have made a lot of progress in clean energy and power, but until companies are willing to make real adjustments, we’re stuck in the stone age pumping our atmosphere full of poisons.

(5) The only way we can force companies to strive for real environmental preservation is if we continue fighting for legislation that compels them to do so. Instead of just asking average citizens to put their water bottles in a recycling bin, we can ask them to also vote with this issue in mind. Elect the right officials who will create and adopt the right legislation that will force companies to act responsibly. The fate of the globe depends on it.
 

Which of these is most likely the author’s purpose for writing this article?

A
To point out that real environmental change will only happen when we hold private companies responsible for their contributions to the problem.
B
To convince people to recycle more often and to look for ways that they can conserve energy.
C
To encourage people to vote for officials who will repeal current laws that were established to protect the environment from private companies.
D
To posit that environmental laws restrict private companies and limit their ability to employ American workers.
Question 12 Explanation: 
Answer choice (A) is correct because it represents the main point the author is trying to make. The author mentions both answer choice (B) and (D), but only in service to their main purpose, represented by choice (A).
Question 13
Saving the Environment

(1) Preserving the environment is a monumental task, and people often have difficulty seeing the connection between what they do and the global problem. Considering how massive the problem is, shaming someone for not recycling a bottle is like yelling at someone for adding a drop of water to a flood. That one bottle is not going to ruin the environment, but 7 billion bottles will. It is certainly important for each citizen to do their part to help improve the world, but too often the onus of environmental preservation is thrust upon the least culpable offenders. Why is our attention so often focused on the daily impact of ordinary citizens rather than the corporations who are doing infinitely more harm every day?

(2) A study called the Carbon Majors Report, conducted in 2015, identified just 100 companies as responsible for 71% of global emissions. These companies are doing far more damage to the environment than average citizens, but America continues to pass legislation that softens environmental protections, allowing companies to continue striving for profit over the health of our globe. While allowing these companies to operate free of restrictions might cause a small uptick in employment, that is a very shortsighted reward considering the consequences.

(3) Governments allow these companies to continue to poison our globe while they demonize the average citizen for their minimal contribution to the problem, or worse: deny that the problem even exists. Problems don’t go away because you ignore them, and until companies are held responsible for their destructive emissions, things are only going to get worse.

(4) Perhaps the most frustrating part of our environmental woes is that the majority of the “problem” companies are invested in industries that could easily be on their way out, like coal and crude oil. Instead of pivoting and investing in new fuels, new energy sources, etc., these companies continue to bank on a dying industry. Scientists have made a lot of progress in clean energy and power, but until companies are willing to make real adjustments, we’re stuck in the stone age pumping our atmosphere full of poisons.

(5) The only way we can force companies to strive for real environmental preservation is if we continue fighting for legislation that compels them to do so. Instead of just asking average citizens to put their water bottles in a recycling bin, we can ask them to also vote with this issue in mind. Elect the right officials who will create and adopt the right legislation that will force companies to act responsibly. The fate of the globe depends on it.
 

Part A

How does the author provide effective support for the main idea of the article?

A
The author asks repeated rhetorical questions to force the reader to think in a unique way.
B
The author presents the problem with the common narrative around environmental protections, then presents what they believe is the real problem.
C
The author compares two equally justified ways of looking at environmental protection and lets the readers decide for themselves.
D
The author appeals to the audience’s emotions with anecdotes and personal stories related to their claim.
Question 13 Explanation: 
Only answer choice (B) reflects the support structure of the passage.
Question 14
Saving the Environment

(1) Preserving the environment is a monumental task, and people often have difficulty seeing the connection between what they do and the global problem. Considering how massive the problem is, shaming someone for not recycling a bottle is like yelling at someone for adding a drop of water to a flood. That one bottle is not going to ruin the environment, but 7 billion bottles will. It is certainly important for each citizen to do their part to help improve the world, but too often the onus of environmental preservation is thrust upon the least culpable offenders. Why is our attention so often focused on the daily impact of ordinary citizens rather than the corporations who are doing infinitely more harm every day?

(2) A study called the Carbon Majors Report, conducted in 2015, identified just 100 companies as responsible for 71% of global emissions. These companies are doing far more damage to the environment than average citizens, but America continues to pass legislation that softens environmental protections, allowing companies to continue striving for profit over the health of our globe. While allowing these companies to operate free of restrictions might cause a small uptick in employment, that is a very shortsighted reward considering the consequences.

(3) Governments allow these companies to continue to poison our globe while they demonize the average citizen for their minimal contribution to the problem, or worse: deny that the problem even exists. Problems don’t go away because you ignore them, and until companies are held responsible for their destructive emissions, things are only going to get worse.

(4) Perhaps the most frustrating part of our environmental woes is that the majority of the “problem” companies are invested in industries that could easily be on their way out, like coal and crude oil. Instead of pivoting and investing in new fuels, new energy sources, etc., these companies continue to bank on a dying industry. Scientists have made a lot of progress in clean energy and power, but until companies are willing to make real adjustments, we’re stuck in the stone age pumping our atmosphere full of poisons.

(5) The only way we can force companies to strive for real environmental preservation is if we continue fighting for legislation that compels them to do so. Instead of just asking average citizens to put their water bottles in a recycling bin, we can ask them to also vote with this issue in mind. Elect the right officials who will create and adopt the right legislation that will force companies to act responsibly. The fate of the globe depends on it.
 

Part B

Which detail from the article best supports the answer to Part A?

A
“Why is our attention so often focused on the daily impact of ordinary citizens rather than the corporations who are doing infinitely more harm every day?”
B
“The fate of the globe depends on it.”
C
“That one bottle is not going to ruin the environment, but 7 billion bottles will.”
D
“It is certainly important for each citizen to do their part to help improve the world, but too often the onus of environmental preservation is thrust upon the least culpable offenders.”
Question 14 Explanation: 
Answer choice (D) is correct because it accurately displays the author’s criticism of common thinking. While answer choice (A) is tempting, it doesn’t support the main thrust of the article as effectively as answer choice (D).
Question 15
Saving the Environment

(1) Preserving the environment is a monumental task, and people often have difficulty seeing the connection between what they do and the global problem. Considering how massive the problem is, shaming someone for not recycling a bottle is like yelling at someone for adding a drop of water to a flood. That one bottle is not going to ruin the environment, but 7 billion bottles will. It is certainly important for each citizen to do their part to help improve the world, but too often the onus of environmental preservation is thrust upon the least culpable offenders. Why is our attention so often focused on the daily impact of ordinary citizens rather than the corporations who are doing infinitely more harm every day?

(2) A study called the Carbon Majors Report, conducted in 2015, identified just 100 companies as responsible for 71% of global emissions. These companies are doing far more damage to the environment than average citizens, but America continues to pass legislation that softens environmental protections, allowing companies to continue striving for profit over the health of our globe. While allowing these companies to operate free of restrictions might cause a small uptick in employment, that is a very shortsighted reward considering the consequences.

(3) Governments allow these companies to continue to poison our globe while they demonize the average citizen for their minimal contribution to the problem, or worse: deny that the problem even exists. Problems don’t go away because you ignore them, and until companies are held responsible for their destructive emissions, things are only going to get worse.

(4) Perhaps the most frustrating part of our environmental woes is that the majority of the “problem” companies are invested in industries that could easily be on their way out, like coal and crude oil. Instead of pivoting and investing in new fuels, new energy sources, etc., these companies continue to bank on a dying industry. Scientists have made a lot of progress in clean energy and power, but until companies are willing to make real adjustments, we’re stuck in the stone age pumping our atmosphere full of poisons.

(5) The only way we can force companies to strive for real environmental preservation is if we continue fighting for legislation that compels them to do so. Instead of just asking average citizens to put their water bottles in a recycling bin, we can ask them to also vote with this issue in mind. Elect the right officials who will create and adopt the right legislation that will force companies to act responsibly. The fate of the globe depends on it.
 

Which of the following statements best represents an inference that can be made from the text?

A
Environmental destruction has been reduced in recent years, though not quickly enough.
B
The American government is trailing most other developed countries in reducing the causes of climate change.
C
The American government is moving in the wrong direction when it comes to protecting the environment.
D
Recycling is a waste of time because the change that can be made from recycling is too minimal to matter.
Question 15 Explanation: 
Answer choice (C) is correct because the author mentions that the American government has recently recalled environmental protections in order to give private companies more freedom to operate as they see fit. None of the other answer choices are supported by the passage, and answer choice (D) is even directly refuted by the author.
Question 16
Saving the Environment

(1) Preserving the environment is a monumental task, and people often have difficulty seeing the connection between what they do and the global problem. Considering how massive the problem is, shaming someone for not recycling a bottle is like yelling at someone for adding a drop of water to a flood. That one bottle is not going to ruin the environment, but 7 billion bottles will. It is certainly important for each citizen to do their part to help improve the world, but too often the onus of environmental preservation is thrust upon the least culpable offenders. Why is our attention so often focused on the daily impact of ordinary citizens rather than the corporations who are doing infinitely more harm every day?

(2) A study called the Carbon Majors Report, conducted in 2015, identified just 100 companies as responsible for 71% of global emissions. These companies are doing far more damage to the environment than average citizens, but America continues to pass legislation that softens environmental protections, allowing companies to continue striving for profit over the health of our globe. While allowing these companies to operate free of restrictions might cause a small uptick in employment, that is a very shortsighted reward considering the consequences.

(3) Governments allow these companies to continue to poison our globe while they demonize the average citizen for their minimal contribution to the problem, or worse: deny that the problem even exists. Problems don’t go away because you ignore them, and until companies are held responsible for their destructive emissions, things are only going to get worse.

(4) Perhaps the most frustrating part of our environmental woes is that the majority of the “problem” companies are invested in industries that could easily be on their way out, like coal and crude oil. Instead of pivoting and investing in new fuels, new energy sources, etc., these companies continue to bank on a dying industry. Scientists have made a lot of progress in clean energy and power, but until companies are willing to make real adjustments, we’re stuck in the stone age pumping our atmosphere full of poisons.

(5) The only way we can force companies to strive for real environmental preservation is if we continue fighting for legislation that compels them to do so. Instead of just asking average citizens to put their water bottles in a recycling bin, we can ask them to also vote with this issue in mind. Elect the right officials who will create and adopt the right legislation that will force companies to act responsibly. The fate of the globe depends on it.
 

Read the sentence from Paragraph 1.

Considering how massive the problem is, shaming someone for not recycling a bottle is like yelling at someone for adding a drop of water to a flood.

This is an example of which of the following?

A
Simile
B
Metaphor
C
Symbolism
D
Allusion
Question 16 Explanation: 
In the quote, the author emphasizes their point by making a comparison using “like.” This simile stresses the author’s idea that demonizing individual citizens is illogical if we aren’t also legislating big companies.
Question 17
Saving the Environment

(1) Preserving the environment is a monumental task, and people often have difficulty seeing the connection between what they do and the global problem. Considering how massive the problem is, shaming someone for not recycling a bottle is like yelling at someone for adding a drop of water to a flood. That one bottle is not going to ruin the environment, but 7 billion bottles will. It is certainly important for each citizen to do their part to help improve the world, but too often the onus of environmental preservation is thrust upon the least culpable offenders. Why is our attention so often focused on the daily impact of ordinary citizens rather than the corporations who are doing infinitely more harm every day?

(2) A study called the Carbon Majors Report, conducted in 2015, identified just 100 companies as responsible for 71% of global emissions. These companies are doing far more damage to the environment than average citizens, but America continues to pass legislation that softens environmental protections, allowing companies to continue striving for profit over the health of our globe. While allowing these companies to operate free of restrictions might cause a small uptick in employment, that is a very shortsighted reward considering the consequences.

(3) Governments allow these companies to continue to poison our globe while they demonize the average citizen for their minimal contribution to the problem, or worse: deny that the problem even exists. Problems don’t go away because you ignore them, and until companies are held responsible for their destructive emissions, things are only going to get worse.

(4) Perhaps the most frustrating part of our environmental woes is that the majority of the “problem” companies are invested in industries that could easily be on their way out, like coal and crude oil. Instead of pivoting and investing in new fuels, new energy sources, etc., these companies continue to bank on a dying industry. Scientists have made a lot of progress in clean energy and power, but until companies are willing to make real adjustments, we’re stuck in the stone age pumping our atmosphere full of poisons.

(5) The only way we can force companies to strive for real environmental preservation is if we continue fighting for legislation that compels them to do so. Instead of just asking average citizens to put their water bottles in a recycling bin, we can ask them to also vote with this issue in mind. Elect the right officials who will create and adopt the right legislation that will force companies to act responsibly. The fate of the globe depends on it.
 

Read the sentence from Paragraph 3.

Problems don’t go away because you ignore them, and until companies are held responsible for their destructive emissions, things are only going to get worse.

Which key idea does the sentence support?

A
Average citizens need to care more about making a change.
B
People who work for these companies are heartless and short-sighted.
C
Private companies want to fix the problem; they just need realistic solutions that they can explore.
D
Private companies are not going to change as long as we allow them to choose profit over responsibility.
Question 17 Explanation: 
Answer choice (D) is correct because the sentence provided uses the phrase “until companies are held responsible,” which shows that companies are not willing to change on their own. This does not mean that everyone who works at these companies is evil (B), but it should take pressure off individual citizens (A).
Question 18
Saving the Environment

(1) Preserving the environment is a monumental task, and people often have difficulty seeing the connection between what they do and the global problem. Considering how massive the problem is, shaming someone for not recycling a bottle is like yelling at someone for adding a drop of water to a flood. That one bottle is not going to ruin the environment, but 7 billion bottles will. It is certainly important for each citizen to do their part to help improve the world, but too often the onus of environmental preservation is thrust upon the least culpable offenders. Why is our attention so often focused on the daily impact of ordinary citizens rather than the corporations who are doing infinitely more harm every day?

(2) A study called the Carbon Majors Report, conducted in 2015, identified just 100 companies as responsible for 71% of global emissions. These companies are doing far more damage to the environment than average citizens, but America continues to pass legislation that softens environmental protections, allowing companies to continue striving for profit over the health of our globe. While allowing these companies to operate free of restrictions might cause a small uptick in employment, that is a very shortsighted reward considering the consequences.

(3) Governments allow these companies to continue to poison our globe while they demonize the average citizen for their minimal contribution to the problem, or worse: deny that the problem even exists. Problems don’t go away because you ignore them, and until companies are held responsible for their destructive emissions, things are only going to get worse.

(4) Perhaps the most frustrating part of our environmental woes is that the majority of the “problem” companies are invested in industries that could easily be on their way out, like coal and crude oil. Instead of pivoting and investing in new fuels, new energy sources, etc., these companies continue to bank on a dying industry. Scientists have made a lot of progress in clean energy and power, but until companies are willing to make real adjustments, we’re stuck in the stone age pumping our atmosphere full of poisons.

(5) The only way we can force companies to strive for real environmental preservation is if we continue fighting for legislation that compels them to do so. Instead of just asking average citizens to put their water bottles in a recycling bin, we can ask them to also vote with this issue in mind. Elect the right officials who will create and adopt the right legislation that will force companies to act responsibly. The fate of the globe depends on it.
 

Which of the following statements from the article prove that the author has taken the opposing viewpoint into consideration?

A
“While allowing these companies to operate free of restrictions might cause a small uptick in employment, that is a very shortsighted reward considering the consequences.”
B
“A study called the Carbon Majors Report, conducted in 2015, identified just 100 companies as responsible for 71% of global emissions.”
C
“Preserving the environment is a monumental task, and people often have difficulty seeing the connection between what they do and the global problem.”
D
“Perhaps the most frustrating part of our environmental woes is that the majority of the “problem” companies are invested in industries that could easily be on their way out, like coal and crude oil.”
Question 18 Explanation: 
The beginning of the sentence in answer choice (A) considers the motivations of the American government. By writing, “While allowing these companies…”, the author has set up the motivations of the opposition so that they can effectively refute these motivations in the second half of the statement. This is an effective use of a counter-argument.
Question 19
Saving the Environment

(1) Preserving the environment is a monumental task, and people often have difficulty seeing the connection between what they do and the global problem. Considering how massive the problem is, shaming someone for not recycling a bottle is like yelling at someone for adding a drop of water to a flood. That one bottle is not going to ruin the environment, but 7 billion bottles will. It is certainly important for each citizen to do their part to help improve the world, but too often the onus of environmental preservation is thrust upon the least culpable offenders. Why is our attention so often focused on the daily impact of ordinary citizens rather than the corporations who are doing infinitely more harm every day?

(2) A study called the Carbon Majors Report, conducted in 2015, identified just 100 companies as responsible for 71% of global emissions. These companies are doing far more damage to the environment than average citizens, but America continues to pass legislation that softens environmental protections, allowing companies to continue striving for profit over the health of our globe. While allowing these companies to operate free of restrictions might cause a small uptick in employment, that is a very shortsighted reward considering the consequences.

(3) Governments allow these companies to continue to poison our globe while they demonize the average citizen for their minimal contribution to the problem, or worse: deny that the problem even exists. Problems don’t go away because you ignore them, and until companies are held responsible for their destructive emissions, things are only going to get worse.

(4) Perhaps the most frustrating part of our environmental woes is that the majority of the “problem” companies are invested in industries that could easily be on their way out, like coal and crude oil. Instead of pivoting and investing in new fuels, new energy sources, etc., these companies continue to bank on a dying industry. Scientists have made a lot of progress in clean energy and power, but until companies are willing to make real adjustments, we’re stuck in the stone age pumping our atmosphere full of poisons.

(5) The only way we can force companies to strive for real environmental preservation is if we continue fighting for legislation that compels them to do so. Instead of just asking average citizens to put their water bottles in a recycling bin, we can ask them to also vote with this issue in mind. Elect the right officials who will create and adopt the right legislation that will force companies to act responsibly. The fate of the globe depends on it.
 

According to Paragraph 5, how can average citizens make the biggest difference?

A
Gathering together outside of the headquarters of the companies that are the biggest offenders will convince them to change.
B
If they vote for the right officials, those officials can create legislation that will force companies to make the environment a priority.
C
There is nothing that the average citizen can do as long as big companies continue to cause most of the problem.
D
If they recycle in their own homes, and everyone does the same, the impact will be enormous.
Question 19 Explanation: 
The author takes readers through a rather simple sequential strategy in the final paragraph of the article. The author says that it’s important for the average citizen to recycle, but it’s even more important for them to vote mindfully in order to force corporations to do their part.
Question 20
Defining Success

(1) In his 2008 book, Outliers, Malcolm Gladwell challenged our understanding of successful people. We tend to look at successful people, he noted, as freaks of nature, people who are so intelligent, or skilled, or talented that they were destined for success. Gladwell’s book, however, argued that this is a superficial understanding of what it takes to be successful. For someone to be successful, they have to have intelligence, skill, and/or talent. But for someone to find the kind of success that Bill Gates has found, or that The Beatles have found, a lot of other factors have to fall into place. Where you are born, when you are born, your cultural legacy, and a myriad of other factors have to line up for you to truly be what Gladwell refers to as “an outlier.” With examples of people all around us that have benefitted greatly from their circumstances, why do we still like to think of successful people as different than us, people destined for success because of their other-worldly intelligence or talent? The explanation for this mental block we seem to have against understanding what leads a person to success is both simple and tragic.

(2) The real reason we like to look at wildly successful people as “others,” people deserving of a success we could never hope to achieve because we lack whatever they have, is because this makes it easier for us to reconcile our own lack of success. In simpler terms, we would rather think that these people have something we don’t have than that we could have achieved their success under different circumstances. This is really a depressing theory about the way we think, not only because it reveals how far we will go to protect our pride, but also because it directly hinders us from finding the kind of success that we idolize. When we decide that a goal is unattainable right from the start, we more easily justify our own inaction.

(3) Accepting defeat right from the start only ensures that we will never find success. In this sense, Gladwell’s theories are both supportive and discouraging. While Gladwell argues that immensely successful people do not possess some other-worldly abilities, he also posits that all types of factors need to line up for someone to achieve this level of success. In this way, his theory is encouraging because it means that many of us have the potential to achieve great success, but it is discouraging in that our success is still out of our control. In his book, Gladwell discusses Bill Gates as he was growing up. Yes, he was a very intelligent young man, but the level of his success is just as much a product of Gates growing up at a time when computer programming was just becoming possible, living in a place where he had access to a computer that could program, and all the other factors that went right for him. If a child is born right now with all the same inherent intelligence as Bill Gates and potential for computer programming, she isn’t likely to reach his level of success because he was there at the beginning. It’s like the difference between buying stock in Apple right now and buying stock in Apple 30 years ago. Bill Gates was learning programming before anyone realized how valuable it was, and that’s why he isn’t just a good programmer with a good job. What we sometimes forget, though, is that success is not black and white. That good programmer with a good job is successful even if she never reaches the kind of success that Bill Gates has achieved.

(4) Our natural inclination to put things into schemas and turn everything into dichotomous groups keeps us not only from finding success in our lives, but from understanding when we have achieved success. People don’t belong in groups where Bill Gates is successful and anyone who makes less money than him is unsuccessful. There are studies that show that happiness isn’t tied to this kind of success. In a well-publicized study, Angus Deaton and Daniel Kahneman posited that a person’s happiness increases as his income increases, but only until he earns about $75,000 a year; after that, returns begin to diminish. In other words, we can attribute success to income, but is there any indication that Bill Gates is happier than someone with a good job and a good income? Once we start realizing that we can be successful even if we aren’t rich and famous, we will live much happier lives. We may not all have the specific opportunities that someone like Bill Gates has, but we all have the ability to achieve success as long as we maintain a reasonable definition of the word.
 

Read this sentence from Paragraph 3.

Yes, he was a very intelligent young man, but the level of his success is just as much a product of Gates growing up at a time when computer programming was just becoming possible, living in a place where he had access to a computer that could program, and all the other factors that went right for him.

Which key idea does this sentence support?

A
People need to redefine how they understand success.
B
Everyone has the ability to achieve success, even if we can’t achieve the kind of success Bill Gates has reached.
C
Successful people have something innately different about them that makes them more likely to succeed than the rest of us.
D
Gladwell argues that success isn’t just a matter of innate intelligence and ability, it is also a matter of our circumstances.
Question 20 Explanation: 
The correct answer is answer choice (D). While answer choices (A) and (B) are both key ideas that are discussed in the passage, they are not supported by the above sentence. Answer choice (C) is directly refuted by the passage.
Question 21
Defining Success

(1) In his 2008 book, Outliers, Malcolm Gladwell challenged our understanding of successful people. We tend to look at successful people, he noted, as freaks of nature, people who are so intelligent, or skilled, or talented that they were destined for success. Gladwell’s book, however, argued that this is a superficial understanding of what it takes to be successful. For someone to be successful, they have to have intelligence, skill, and/or talent. But for someone to find the kind of success that Bill Gates has found, or that The Beatles have found, a lot of other factors have to fall into place. Where you are born, when you are born, your cultural legacy, and a myriad of other factors have to line up for you to truly be what Gladwell refers to as “an outlier.” With examples of people all around us that have benefitted greatly from their circumstances, why do we still like to think of successful people as different than us, people destined for success because of their other-worldly intelligence or talent? The explanation for this mental block we seem to have against understanding what leads a person to success is both simple and tragic.

(2) The real reason we like to look at wildly successful people as “others,” people deserving of a success we could never hope to achieve because we lack whatever they have, is because this makes it easier for us to reconcile our own lack of success. In simpler terms, we would rather think that these people have something we don’t have than that we could have achieved their success under different circumstances. This is really a depressing theory about the way we think, not only because it reveals how far we will go to protect our pride, but also because it directly hinders us from finding the kind of success that we idolize. When we decide that a goal is unattainable right from the start, we more easily justify our own inaction.

(3) Accepting defeat right from the start only ensures that we will never find success. In this sense, Gladwell’s theories are both supportive and discouraging. While Gladwell argues that immensely successful people do not possess some other-worldly abilities, he also posits that all types of factors need to line up for someone to achieve this level of success. In this way, his theory is encouraging because it means that many of us have the potential to achieve great success, but it is discouraging in that our success is still out of our control. In his book, Gladwell discusses Bill Gates as he was growing up. Yes, he was a very intelligent young man, but the level of his success is just as much a product of Gates growing up at a time when computer programming was just becoming possible, living in a place where he had access to a computer that could program, and all the other factors that went right for him. If a child is born right now with all the same inherent intelligence as Bill Gates and potential for computer programming, she isn’t likely to reach his level of success because he was there at the beginning. It’s like the difference between buying stock in Apple right now and buying stock in Apple 30 years ago. Bill Gates was learning programming before anyone realized how valuable it was, and that’s why he isn’t just a good programmer with a good job. What we sometimes forget, though, is that success is not black and white. That good programmer with a good job is successful even if she never reaches the kind of success that Bill Gates has achieved.

(4) Our natural inclination to put things into schemas and turn everything into dichotomous groups keeps us not only from finding success in our lives, but from understanding when we have achieved success. People don’t belong in groups where Bill Gates is successful and anyone who makes less money than him is unsuccessful. There are studies that show that happiness isn’t tied to this kind of success. In a well-publicized study, Angus Deaton and Daniel Kahneman posited that a person’s happiness increases as his income increases, but only until he earns about $75,000 a year; after that, returns begin to diminish. In other words, we can attribute success to income, but is there any indication that Bill Gates is happier than someone with a good job and a good income? Once we start realizing that we can be successful even if we aren’t rich and famous, we will live much happier lives. We may not all have the specific opportunities that someone like Bill Gates has, but we all have the ability to achieve success as long as we maintain a reasonable definition of the word.
 

Which of these is most likely the author’s purpose for writing this passage?

A
To convince people that Malcolm Gladwell’s theories on success are true.
B
To encourage people to let their pride go and strive for success.
C
To refute Malcolm Gladwell’s theories on success and how we achieve it.
D
To convince people that money does not bring people happiness.
Question 21 Explanation: 
The correct answer is answer choice (B). While the writer does not refute Gladwell’s theories, he is also not trying to convince people that they are accurate, which makes answer choices (A) and (C) incorrect. Similarly, the writer tries to convince people that money isn’t the only thing that brings happiness, he acknowledges that to a point, money does bring more happiness. This makes answer choice (D) incorrect.
Question 22
Defining Success

(1) In his 2008 book, Outliers, Malcolm Gladwell challenged our understanding of successful people. We tend to look at successful people, he noted, as freaks of nature, people who are so intelligent, or skilled, or talented that they were destined for success. Gladwell’s book, however, argued that this is a superficial understanding of what it takes to be successful. For someone to be successful, they have to have intelligence, skill, and/or talent. But for someone to find the kind of success that Bill Gates has found, or that The Beatles have found, a lot of other factors have to fall into place. Where you are born, when you are born, your cultural legacy, and a myriad of other factors have to line up for you to truly be what Gladwell refers to as “an outlier.” With examples of people all around us that have benefitted greatly from their circumstances, why do we still like to think of successful people as different than us, people destined for success because of their other-worldly intelligence or talent? The explanation for this mental block we seem to have against understanding what leads a person to success is both simple and tragic.

(2) The real reason we like to look at wildly successful people as “others,” people deserving of a success we could never hope to achieve because we lack whatever they have, is because this makes it easier for us to reconcile our own lack of success. In simpler terms, we would rather think that these people have something we don’t have than that we could have achieved their success under different circumstances. This is really a depressing theory about the way we think, not only because it reveals how far we will go to protect our pride, but also because it directly hinders us from finding the kind of success that we idolize. When we decide that a goal is unattainable right from the start, we more easily justify our own inaction.

(3) Accepting defeat right from the start only ensures that we will never find success. In this sense, Gladwell’s theories are both supportive and discouraging. While Gladwell argues that immensely successful people do not possess some other-worldly abilities, he also posits that all types of factors need to line up for someone to achieve this level of success. In this way, his theory is encouraging because it means that many of us have the potential to achieve great success, but it is discouraging in that our success is still out of our control. In his book, Gladwell discusses Bill Gates as he was growing up. Yes, he was a very intelligent young man, but the level of his success is just as much a product of Gates growing up at a time when computer programming was just becoming possible, living in a place where he had access to a computer that could program, and all the other factors that went right for him. If a child is born right now with all the same inherent intelligence as Bill Gates and potential for computer programming, she isn’t likely to reach his level of success because he was there at the beginning. It’s like the difference between buying stock in Apple right now and buying stock in Apple 30 years ago. Bill Gates was learning programming before anyone realized how valuable it was, and that’s why he isn’t just a good programmer with a good job. What we sometimes forget, though, is that success is not black and white. That good programmer with a good job is successful even if she never reaches the kind of success that Bill Gates has achieved.

(4) Our natural inclination to put things into schemas and turn everything into dichotomous groups keeps us not only from finding success in our lives, but from understanding when we have achieved success. People don’t belong in groups where Bill Gates is successful and anyone who makes less money than him is unsuccessful. There are studies that show that happiness isn’t tied to this kind of success. In a well-publicized study, Angus Deaton and Daniel Kahneman posited that a person’s happiness increases as his income increases, but only until he earns about $75,000 a year; after that, returns begin to diminish. In other words, we can attribute success to income, but is there any indication that Bill Gates is happier than someone with a good job and a good income? Once we start realizing that we can be successful even if we aren’t rich and famous, we will live much happier lives. We may not all have the specific opportunities that someone like Bill Gates has, but we all have the ability to achieve success as long as we maintain a reasonable definition of the word.
 

Part A

How does the writer provide effective support for the main idea of the passage?

A
The writer references respected books and economic studies.
B
The writer appeals to the readers emotions with a moving story.
C
The writer compares Gladwell’s book to the study done by Deaton and Kahneman.
D
The writer provides logical steps people can follow to find success.
Question 22 Explanation: 
The correct answer is answer choice (A). The writer uses Malcolm Gladwell’s book, Outliers, as an initiation into his own argument, and uses the study done by Deaton and Kahneman to support his ideas. None of the other answer choices are supported by the passage.
Question 23
Defining Success

(1) In his 2008 book, Outliers, Malcolm Gladwell challenged our understanding of successful people. We tend to look at successful people, he noted, as freaks of nature, people who are so intelligent, or skilled, or talented that they were destined for success. Gladwell’s book, however, argued that this is a superficial understanding of what it takes to be successful. For someone to be successful, they have to have intelligence, skill, and/or talent. But for someone to find the kind of success that Bill Gates has found, or that The Beatles have found, a lot of other factors have to fall into place. Where you are born, when you are born, your cultural legacy, and a myriad of other factors have to line up for you to truly be what Gladwell refers to as “an outlier.” With examples of people all around us that have benefitted greatly from their circumstances, why do we still like to think of successful people as different than us, people destined for success because of their other-worldly intelligence or talent? The explanation for this mental block we seem to have against understanding what leads a person to success is both simple and tragic.

(2) The real reason we like to look at wildly successful people as “others,” people deserving of a success we could never hope to achieve because we lack whatever they have, is because this makes it easier for us to reconcile our own lack of success. In simpler terms, we would rather think that these people have something we don’t have than that we could have achieved their success under different circumstances. This is really a depressing theory about the way we think, not only because it reveals how far we will go to protect our pride, but also because it directly hinders us from finding the kind of success that we idolize. When we decide that a goal is unattainable right from the start, we more easily justify our own inaction.

(3) Accepting defeat right from the start only ensures that we will never find success. In this sense, Gladwell’s theories are both supportive and discouraging. While Gladwell argues that immensely successful people do not possess some other-worldly abilities, he also posits that all types of factors need to line up for someone to achieve this level of success. In this way, his theory is encouraging because it means that many of us have the potential to achieve great success, but it is discouraging in that our success is still out of our control. In his book, Gladwell discusses Bill Gates as he was growing up. Yes, he was a very intelligent young man, but the level of his success is just as much a product of Gates growing up at a time when computer programming was just becoming possible, living in a place where he had access to a computer that could program, and all the other factors that went right for him. If a child is born right now with all the same inherent intelligence as Bill Gates and potential for computer programming, she isn’t likely to reach his level of success because he was there at the beginning. It’s like the difference between buying stock in Apple right now and buying stock in Apple 30 years ago. Bill Gates was learning programming before anyone realized how valuable it was, and that’s why he isn’t just a good programmer with a good job. What we sometimes forget, though, is that success is not black and white. That good programmer with a good job is successful even if she never reaches the kind of success that Bill Gates has achieved.

(4) Our natural inclination to put things into schemas and turn everything into dichotomous groups keeps us not only from finding success in our lives, but from understanding when we have achieved success. People don’t belong in groups where Bill Gates is successful and anyone who makes less money than him is unsuccessful. There are studies that show that happiness isn’t tied to this kind of success. In a well-publicized study, Angus Deaton and Daniel Kahneman posited that a person’s happiness increases as his income increases, but only until he earns about $75,000 a year; after that, returns begin to diminish. In other words, we can attribute success to income, but is there any indication that Bill Gates is happier than someone with a good job and a good income? Once we start realizing that we can be successful even if we aren’t rich and famous, we will live much happier lives. We may not all have the specific opportunities that someone like Bill Gates has, but we all have the ability to achieve success as long as we maintain a reasonable definition of the word.
 

Part B

Which sentence from the passage best supports the answer to Part A?

A
“The real reason we like to look at wildly successful people as “others,” people deserving of a success we could never hope to achieve because we lack whatever they have, is because this makes it easier for us to reconcile our own lack of success.”
B
“That good programmer with a good job is successful even if she never reaches the kind of success that Bill Gates has achieved.”
C
“Gladwell’s book, however, argued that this is a superficial understanding of what it takes to be successful.”
D
“Our natural inclination to put things into schemas and turn everything into dichotomous groups keeps us not only from finding success in our lives, but from understanding when we have achieved success.”
Question 23 Explanation: 
The correct answer to Part B is answer choice (C). This is the only sentence form the passage that directly reflects the writer’s discussion of “respected books and economic studies.”
Question 24
Defining Success

(1) In his 2008 book, Outliers, Malcolm Gladwell challenged our understanding of successful people. We tend to look at successful people, he noted, as freaks of nature, people who are so intelligent, or skilled, or talented that they were destined for success. Gladwell’s book, however, argued that this is a superficial understanding of what it takes to be successful. For someone to be successful, they have to have intelligence, skill, and/or talent. But for someone to find the kind of success that Bill Gates has found, or that The Beatles have found, a lot of other factors have to fall into place. Where you are born, when you are born, your cultural legacy, and a myriad of other factors have to line up for you to truly be what Gladwell refers to as “an outlier.” With examples of people all around us that have benefitted greatly from their circumstances, why do we still like to think of successful people as different than us, people destined for success because of their other-worldly intelligence or talent? The explanation for this mental block we seem to have against understanding what leads a person to success is both simple and tragic.

(2) The real reason we like to look at wildly successful people as “others,” people deserving of a success we could never hope to achieve because we lack whatever they have, is because this makes it easier for us to reconcile our own lack of success. In simpler terms, we would rather think that these people have something we don’t have than that we could have achieved their success under different circumstances. This is really a depressing theory about the way we think, not only because it reveals how far we will go to protect our pride, but also because it directly hinders us from finding the kind of success that we idolize. When we decide that a goal is unattainable right from the start, we more easily justify our own inaction.

(3) Accepting defeat right from the start only ensures that we will never find success. In this sense, Gladwell’s theories are both supportive and discouraging. While Gladwell argues that immensely successful people do not possess some other-worldly abilities, he also posits that all types of factors need to line up for someone to achieve this level of success. In this way, his theory is encouraging because it means that many of us have the potential to achieve great success, but it is discouraging in that our success is still out of our control. In his book, Gladwell discusses Bill Gates as he was growing up. Yes, he was a very intelligent young man, but the level of his success is just as much a product of Gates growing up at a time when computer programming was just becoming possible, living in a place where he had access to a computer that could program, and all the other factors that went right for him. If a child is born right now with all the same inherent intelligence as Bill Gates and potential for computer programming, she isn’t likely to reach his level of success because he was there at the beginning. It’s like the difference between buying stock in Apple right now and buying stock in Apple 30 years ago. Bill Gates was learning programming before anyone realized how valuable it was, and that’s why he isn’t just a good programmer with a good job. What we sometimes forget, though, is that success is not black and white. That good programmer with a good job is successful even if she never reaches the kind of success that Bill Gates has achieved.

(4) Our natural inclination to put things into schemas and turn everything into dichotomous groups keeps us not only from finding success in our lives, but from understanding when we have achieved success. People don’t belong in groups where Bill Gates is successful and anyone who makes less money than him is unsuccessful. There are studies that show that happiness isn’t tied to this kind of success. In a well-publicized study, Angus Deaton and Daniel Kahneman posited that a person’s happiness increases as his income increases, but only until he earns about $75,000 a year; after that, returns begin to diminish. In other words, we can attribute success to income, but is there any indication that Bill Gates is happier than someone with a good job and a good income? Once we start realizing that we can be successful even if we aren’t rich and famous, we will live much happier lives. We may not all have the specific opportunities that someone like Bill Gates has, but we all have the ability to achieve success as long as we maintain a reasonable definition of the word.
 

Which of the following statements best describes the structure of the passage?

A
The writer presents an opinion in the first paragraph and refutes it throughout the rest of the passage.
B
The writer introduces a problem in the first paragraph and explores a potential solution throughout the rest of the passage.
C
The writer introduces a book in the first paragraph and explores its impact on society throughout the rest of the passage.
D
The writer presents a balanced exploration of two sides of an ongoing argument.
Question 24 Explanation: 
The correct answer is answer choice (B). The writer introduces Gladwell’s book in the first paragraph to help illustrate a key idea about why people view success the way they do (problem-solution). Answer choice (C) is incorrect because the writer doesn’t focus the entire passage on the book, and answer choice (D) is incorrect because the writer clearly takes a position. Answer choice (A) might be tempting because when the writer offers a potential solution to a problem, it could seem like he is refuting an idea. He is not refuting it, he is simply offering a potential solution.
Question 25
Defining Success

(1) In his 2008 book, Outliers, Malcolm Gladwell challenged our understanding of successful people. We tend to look at successful people, he noted, as freaks of nature, people who are so intelligent, or skilled, or talented that they were destined for success. Gladwell’s book, however, argued that this is a superficial understanding of what it takes to be successful. For someone to be successful, they have to have intelligence, skill, and/or talent. But for someone to find the kind of success that Bill Gates has found, or that The Beatles have found, a lot of other factors have to fall into place. Where you are born, when you are born, your cultural legacy, and a myriad of other factors have to line up for you to truly be what Gladwell refers to as “an outlier.” With examples of people all around us that have benefitted greatly from their circumstances, why do we still like to think of successful people as different than us, people destined for success because of their other-worldly intelligence or talent? The explanation for this mental block we seem to have against understanding what leads a person to success is both simple and tragic.

(2) The real reason we like to look at wildly successful people as “others,” people deserving of a success we could never hope to achieve because we lack whatever they have, is because this makes it easier for us to reconcile our own lack of success. In simpler terms, we would rather think that these people have something we don’t have than that we could have achieved their success under different circumstances. This is really a depressing theory about the way we think, not only because it reveals how far we will go to protect our pride, but also because it directly hinders us from finding the kind of success that we idolize. When we decide that a goal is unattainable right from the start, we more easily justify our own inaction.

(3) Accepting defeat right from the start only ensures that we will never find success. In this sense, Gladwell’s theories are both supportive and discouraging. While Gladwell argues that immensely successful people do not possess some other-worldly abilities, he also posits that all types of factors need to line up for someone to achieve this level of success. In this way, his theory is encouraging because it means that many of us have the potential to achieve great success, but it is discouraging in that our success is still out of our control. In his book, Gladwell discusses Bill Gates as he was growing up. Yes, he was a very intelligent young man, but the level of his success is just as much a product of Gates growing up at a time when computer programming was just becoming possible, living in a place where he had access to a computer that could program, and all the other factors that went right for him. If a child is born right now with all the same inherent intelligence as Bill Gates and potential for computer programming, she isn’t likely to reach his level of success because he was there at the beginning. It’s like the difference between buying stock in Apple right now and buying stock in Apple 30 years ago. Bill Gates was learning programming before anyone realized how valuable it was, and that’s why he isn’t just a good programmer with a good job. What we sometimes forget, though, is that success is not black and white. That good programmer with a good job is successful even if she never reaches the kind of success that Bill Gates has achieved.

(4) Our natural inclination to put things into schemas and turn everything into dichotomous groups keeps us not only from finding success in our lives, but from understanding when we have achieved success. People don’t belong in groups where Bill Gates is successful and anyone who makes less money than him is unsuccessful. There are studies that show that happiness isn’t tied to this kind of success. In a well-publicized study, Angus Deaton and Daniel Kahneman posited that a person’s happiness increases as his income increases, but only until he earns about $75,000 a year; after that, returns begin to diminish. In other words, we can attribute success to income, but is there any indication that Bill Gates is happier than someone with a good job and a good income? Once we start realizing that we can be successful even if we aren’t rich and famous, we will live much happier lives. We may not all have the specific opportunities that someone like Bill Gates has, but we all have the ability to achieve success as long as we maintain a reasonable definition of the word.
 

Which statement represents a claim made by the writer in paragraph 3?

A
Bill Gates became so successful because he was born in the right place at the right time.
B
Gladwell’s theories about success should only make people feel encouraged.
C
Once people accept that they will never be Bill Gates, they can find real success.
D
Gladwell’s theories about success should only make people feel discouraged.
Question 25 Explanation: 
The correct answer is answer choice (A). In this paragraph, the writer explores Gladwell’s theory that Bill Gates was so successful because of the opportunities that where and when he was born afforded him. He writes that this theory should both encourage and discourage people, which eliminates answer choices (B) and (D) because they are too restrictive. While the writer eventually suggests that people stop worrying about being Bill Gates and start looking at the real successes in their own lives, that isn’t reflected in paragraph 3. In fact, this paragraph may suggest the opposite.
Question 26
Defining Success

(1) In his 2008 book, Outliers, Malcolm Gladwell challenged our understanding of successful people. We tend to look at successful people, he noted, as freaks of nature, people who are so intelligent, or skilled, or talented that they were destined for success. Gladwell’s book, however, argued that this is a superficial understanding of what it takes to be successful. For someone to be successful, they have to have intelligence, skill, and/or talent. But for someone to find the kind of success that Bill Gates has found, or that The Beatles have found, a lot of other factors have to fall into place. Where you are born, when you are born, your cultural legacy, and a myriad of other factors have to line up for you to truly be what Gladwell refers to as “an outlier.” With examples of people all around us that have benefitted greatly from their circumstances, why do we still like to think of successful people as different than us, people destined for success because of their other-worldly intelligence or talent? The explanation for this mental block we seem to have against understanding what leads a person to success is both simple and tragic.

(2) The real reason we like to look at wildly successful people as “others,” people deserving of a success we could never hope to achieve because we lack whatever they have, is because this makes it easier for us to reconcile our own lack of success. In simpler terms, we would rather think that these people have something we don’t have than that we could have achieved their success under different circumstances. This is really a depressing theory about the way we think, not only because it reveals how far we will go to protect our pride, but also because it directly hinders us from finding the kind of success that we idolize. When we decide that a goal is unattainable right from the start, we more easily justify our own inaction.

(3) Accepting defeat right from the start only ensures that we will never find success. In this sense, Gladwell’s theories are both supportive and discouraging. While Gladwell argues that immensely successful people do not possess some other-worldly abilities, he also posits that all types of factors need to line up for someone to achieve this level of success. In this way, his theory is encouraging because it means that many of us have the potential to achieve great success, but it is discouraging in that our success is still out of our control. In his book, Gladwell discusses Bill Gates as he was growing up. Yes, he was a very intelligent young man, but the level of his success is just as much a product of Gates growing up at a time when computer programming was just becoming possible, living in a place where he had access to a computer that could program, and all the other factors that went right for him. If a child is born right now with all the same inherent intelligence as Bill Gates and potential for computer programming, she isn’t likely to reach his level of success because he was there at the beginning. It’s like the difference between buying stock in Apple right now and buying stock in Apple 30 years ago. Bill Gates was learning programming before anyone realized how valuable it was, and that’s why he isn’t just a good programmer with a good job. What we sometimes forget, though, is that success is not black and white. That good programmer with a good job is successful even if she never reaches the kind of success that Bill Gates has achieved.

(4) Our natural inclination to put things into schemas and turn everything into dichotomous groups keeps us not only from finding success in our lives, but from understanding when we have achieved success. People don’t belong in groups where Bill Gates is successful and anyone who makes less money than him is unsuccessful. There are studies that show that happiness isn’t tied to this kind of success. In a well-publicized study, Angus Deaton and Daniel Kahneman posited that a person’s happiness increases as his income increases, but only until he earns about $75,000 a year; after that, returns begin to diminish. In other words, we can attribute success to income, but is there any indication that Bill Gates is happier than someone with a good job and a good income? Once we start realizing that we can be successful even if we aren’t rich and famous, we will live much happier lives. We may not all have the specific opportunities that someone like Bill Gates has, but we all have the ability to achieve success as long as we maintain a reasonable definition of the word.
 

Which sentence from paragraph 3 best supports the answer from Question 5?

A
“Accepting defeat right from the start only ensures that we will never find success.”
B
“What we sometimes forget, though, is that success is not black and white.”
C
“Yes, he was a very intelligent young man, but the level of his success is just as much a product of Gates growing up at a time when computer programming was just becoming possible, living in a place where he had access to a computer that could program, and all the other factors that went right for him.”
D
“That good programmer with a good job is successful even if she never reaches the kind of success that Bill Gates has achieved.”
Question 26 Explanation: 
The correct answer is answer choice (C). Answer choice (D) might be tempting, but it actually represents the next step in the writer’s logic, not support for the answer to Question 5. Answer choice (A) is an introductory sentence and therefore not supporting, while answer choice (B) more accurately works as a transition between ideas in the paragraph.
Question 27
Defining Success

(1) In his 2008 book, Outliers, Malcolm Gladwell challenged our understanding of successful people. We tend to look at successful people, he noted, as freaks of nature, people who are so intelligent, or skilled, or talented that they were destined for success. Gladwell’s book, however, argued that this is a superficial understanding of what it takes to be successful. For someone to be successful, they have to have intelligence, skill, and/or talent. But for someone to find the kind of success that Bill Gates has found, or that The Beatles have found, a lot of other factors have to fall into place. Where you are born, when you are born, your cultural legacy, and a myriad of other factors have to line up for you to truly be what Gladwell refers to as “an outlier.” With examples of people all around us that have benefitted greatly from their circumstances, why do we still like to think of successful people as different than us, people destined for success because of their other-worldly intelligence or talent? The explanation for this mental block we seem to have against understanding what leads a person to success is both simple and tragic.

(2) The real reason we like to look at wildly successful people as “others,” people deserving of a success we could never hope to achieve because we lack whatever they have, is because this makes it easier for us to reconcile our own lack of success. In simpler terms, we would rather think that these people have something we don’t have than that we could have achieved their success under different circumstances. This is really a depressing theory about the way we think, not only because it reveals how far we will go to protect our pride, but also because it directly hinders us from finding the kind of success that we idolize. When we decide that a goal is unattainable right from the start, we more easily justify our own inaction.

(3) Accepting defeat right from the start only ensures that we will never find success. In this sense, Gladwell’s theories are both supportive and discouraging. While Gladwell argues that immensely successful people do not possess some other-worldly abilities, he also posits that all types of factors need to line up for someone to achieve this level of success. In this way, his theory is encouraging because it means that many of us have the potential to achieve great success, but it is discouraging in that our success is still out of our control. In his book, Gladwell discusses Bill Gates as he was growing up. Yes, he was a very intelligent young man, but the level of his success is just as much a product of Gates growing up at a time when computer programming was just becoming possible, living in a place where he had access to a computer that could program, and all the other factors that went right for him. If a child is born right now with all the same inherent intelligence as Bill Gates and potential for computer programming, she isn’t likely to reach his level of success because he was there at the beginning. It’s like the difference between buying stock in Apple right now and buying stock in Apple 30 years ago. Bill Gates was learning programming before anyone realized how valuable it was, and that’s why he isn’t just a good programmer with a good job. What we sometimes forget, though, is that success is not black and white. That good programmer with a good job is successful even if she never reaches the kind of success that Bill Gates has achieved.

(4) Our natural inclination to put things into schemas and turn everything into dichotomous groups keeps us not only from finding success in our lives, but from understanding when we have achieved success. People don’t belong in groups where Bill Gates is successful and anyone who makes less money than him is unsuccessful. There are studies that show that happiness isn’t tied to this kind of success. In a well-publicized study, Angus Deaton and Daniel Kahneman posited that a person’s happiness increases as his income increases, but only until he earns about $75,000 a year; after that, returns begin to diminish. In other words, we can attribute success to income, but is there any indication that Bill Gates is happier than someone with a good job and a good income? Once we start realizing that we can be successful even if we aren’t rich and famous, we will live much happier lives. We may not all have the specific opportunities that someone like Bill Gates has, but we all have the ability to achieve success as long as we maintain a reasonable definition of the word.
 

Read this sentence from the passage.

Where you are born, when you are born, your cultural legacy, and a myriad of other factors have to line up for you to truly be what Gladwell refers to as “an outlier.”

Which of the following words/phrases best represents the meaning of myriad in the sentence above?

A
Scarcity
B
Abundance
C
Medley
D
Spectrum
Question 27 Explanation: 
The correct answer is answer choice (B). In the sentence, the writer uses myriad to mean a lot, or an abundance, of factors. Answer choices (C) and (D) might be tempting because these factors could be varied and different. However, myriad only technically refers to the number of factors. Answer choice (A) is the opposite of myriad.
Question 28
Defining Success

(1) In his 2008 book, Outliers, Malcolm Gladwell challenged our understanding of successful people. We tend to look at successful people, he noted, as freaks of nature, people who are so intelligent, or skilled, or talented that they were destined for success. Gladwell’s book, however, argued that this is a superficial understanding of what it takes to be successful. For someone to be successful, they have to have intelligence, skill, and/or talent. But for someone to find the kind of success that Bill Gates has found, or that The Beatles have found, a lot of other factors have to fall into place. Where you are born, when you are born, your cultural legacy, and a myriad of other factors have to line up for you to truly be what Gladwell refers to as “an outlier.” With examples of people all around us that have benefitted greatly from their circumstances, why do we still like to think of successful people as different than us, people destined for success because of their other-worldly intelligence or talent? The explanation for this mental block we seem to have against understanding what leads a person to success is both simple and tragic.

(2) The real reason we like to look at wildly successful people as “others,” people deserving of a success we could never hope to achieve because we lack whatever they have, is because this makes it easier for us to reconcile our own lack of success. In simpler terms, we would rather think that these people have something we don’t have than that we could have achieved their success under different circumstances. This is really a depressing theory about the way we think, not only because it reveals how far we will go to protect our pride, but also because it directly hinders us from finding the kind of success that we idolize. When we decide that a goal is unattainable right from the start, we more easily justify our own inaction.

(3) Accepting defeat right from the start only ensures that we will never find success. In this sense, Gladwell’s theories are both supportive and discouraging. While Gladwell argues that immensely successful people do not possess some other-worldly abilities, he also posits that all types of factors need to line up for someone to achieve this level of success. In this way, his theory is encouraging because it means that many of us have the potential to achieve great success, but it is discouraging in that our success is still out of our control. In his book, Gladwell discusses Bill Gates as he was growing up. Yes, he was a very intelligent young man, but the level of his success is just as much a product of Gates growing up at a time when computer programming was just becoming possible, living in a place where he had access to a computer that could program, and all the other factors that went right for him. If a child is born right now with all the same inherent intelligence as Bill Gates and potential for computer programming, she isn’t likely to reach his level of success because he was there at the beginning. It’s like the difference between buying stock in Apple right now and buying stock in Apple 30 years ago. Bill Gates was learning programming before anyone realized how valuable it was, and that’s why he isn’t just a good programmer with a good job. What we sometimes forget, though, is that success is not black and white. That good programmer with a good job is successful even if she never reaches the kind of success that Bill Gates has achieved.

(4) Our natural inclination to put things into schemas and turn everything into dichotomous groups keeps us not only from finding success in our lives, but from understanding when we have achieved success. People don’t belong in groups where Bill Gates is successful and anyone who makes less money than him is unsuccessful. There are studies that show that happiness isn’t tied to this kind of success. In a well-publicized study, Angus Deaton and Daniel Kahneman posited that a person’s happiness increases as his income increases, but only until he earns about $75,000 a year; after that, returns begin to diminish. In other words, we can attribute success to income, but is there any indication that Bill Gates is happier than someone with a good job and a good income? Once we start realizing that we can be successful even if we aren’t rich and famous, we will live much happier lives. We may not all have the specific opportunities that someone like Bill Gates has, but we all have the ability to achieve success as long as we maintain a reasonable definition of the word.
 

What purpose does the economic study serve in the final paragraph of the passage?

A
It supports the writer’s assertion that we can best calculate success by how much money someone makes.
B
It supports the writer’s assertion that Bill Gates is a happy person.
C
It supports the writer’s assertion that success relies on a lot of factors, most of which are out of our control.
D
It supports the writer’s assertion that money only leads to happiness up to a certain point.
Question 28 Explanation: 
The correct answer is answer choice (D). The study showed that money and happiness are only positively correlated until we start making around $75,000 a year, then the correlation diminishes. Answer choice (C) may have been tempting because it reflects an important idea that the writer explores in the passage, but not one supported by this piece of evidence. Answer choices (A) and (B) are directly refuted by the passage.
Question 29
Defining Success

(1) In his 2008 book, Outliers, Malcolm Gladwell challenged our understanding of successful people. We tend to look at successful people, he noted, as freaks of nature, people who are so intelligent, or skilled, or talented that they were destined for success. Gladwell’s book, however, argued that this is a superficial understanding of what it takes to be successful. For someone to be successful, they have to have intelligence, skill, and/or talent. But for someone to find the kind of success that Bill Gates has found, or that The Beatles have found, a lot of other factors have to fall into place. Where you are born, when you are born, your cultural legacy, and a myriad of other factors have to line up for you to truly be what Gladwell refers to as “an outlier.” With examples of people all around us that have benefitted greatly from their circumstances, why do we still like to think of successful people as different than us, people destined for success because of their other-worldly intelligence or talent? The explanation for this mental block we seem to have against understanding what leads a person to success is both simple and tragic.

(2) The real reason we like to look at wildly successful people as “others,” people deserving of a success we could never hope to achieve because we lack whatever they have, is because this makes it easier for us to reconcile our own lack of success. In simpler terms, we would rather think that these people have something we don’t have than that we could have achieved their success under different circumstances. This is really a depressing theory about the way we think, not only because it reveals how far we will go to protect our pride, but also because it directly hinders us from finding the kind of success that we idolize. When we decide that a goal is unattainable right from the start, we more easily justify our own inaction.

(3) Accepting defeat right from the start only ensures that we will never find success. In this sense, Gladwell’s theories are both supportive and discouraging. While Gladwell argues that immensely successful people do not possess some other-worldly abilities, he also posits that all types of factors need to line up for someone to achieve this level of success. In this way, his theory is encouraging because it means that many of us have the potential to achieve great success, but it is discouraging in that our success is still out of our control. In his book, Gladwell discusses Bill Gates as he was growing up. Yes, he was a very intelligent young man, but the level of his success is just as much a product of Gates growing up at a time when computer programming was just becoming possible, living in a place where he had access to a computer that could program, and all the other factors that went right for him. If a child is born right now with all the same inherent intelligence as Bill Gates and potential for computer programming, she isn’t likely to reach his level of success because he was there at the beginning. It’s like the difference between buying stock in Apple right now and buying stock in Apple 30 years ago. Bill Gates was learning programming before anyone realized how valuable it was, and that’s why he isn’t just a good programmer with a good job. What we sometimes forget, though, is that success is not black and white. That good programmer with a good job is successful even if she never reaches the kind of success that Bill Gates has achieved.

(4) Our natural inclination to put things into schemas and turn everything into dichotomous groups keeps us not only from finding success in our lives, but from understanding when we have achieved success. People don’t belong in groups where Bill Gates is successful and anyone who makes less money than him is unsuccessful. There are studies that show that happiness isn’t tied to this kind of success. In a well-publicized study, Angus Deaton and Daniel Kahneman posited that a person’s happiness increases as his income increases, but only until he earns about $75,000 a year; after that, returns begin to diminish. In other words, we can attribute success to income, but is there any indication that Bill Gates is happier than someone with a good job and a good income? Once we start realizing that we can be successful even if we aren’t rich and famous, we will live much happier lives. We may not all have the specific opportunities that someone like Bill Gates has, but we all have the ability to achieve success as long as we maintain a reasonable definition of the word.
 

Read this sentence from the passage.

Our natural inclination to put things into schemas and turn everything into dichotomous groups keeps us not only from finding success in our lives, but from understanding when we have achieved success.

Which of the following answers best represents the meaning of dichotomous in the above sentence?

A
Bipartisan
B
Simplistic
C
Combative
D
Organized
Question 29 Explanation: 
The correct answer is answer choice (A). The word “dichotomous” means having two opposing sides, or bipartisan. These sides may be combative (C) and organized (D), but that is not what the word means. Answer choice (B) may be tempting because the writer is complaining that this categorization is too simplistic, but that sentiment isn’t reflected by the word “dichotomous,” but instead by the tone of the sentence (and paragraph).
Question 30
Defining Success

(1) In his 2008 book, Outliers, Malcolm Gladwell challenged our understanding of successful people. We tend to look at successful people, he noted, as freaks of nature, people who are so intelligent, or skilled, or talented that they were destined for success. Gladwell’s book, however, argued that this is a superficial understanding of what it takes to be successful. For someone to be successful, they have to have intelligence, skill, and/or talent. But for someone to find the kind of success that Bill Gates has found, or that The Beatles have found, a lot of other factors have to fall into place. Where you are born, when you are born, your cultural legacy, and a myriad of other factors have to line up for you to truly be what Gladwell refers to as “an outlier.” With examples of people all around us that have benefitted greatly from their circumstances, why do we still like to think of successful people as different than us, people destined for success because of their other-worldly intelligence or talent? The explanation for this mental block we seem to have against understanding what leads a person to success is both simple and tragic.

(2) The real reason we like to look at wildly successful people as “others,” people deserving of a success we could never hope to achieve because we lack whatever they have, is because this makes it easier for us to reconcile our own lack of success. In simpler terms, we would rather think that these people have something we don’t have than that we could have achieved their success under different circumstances. This is really a depressing theory about the way we think, not only because it reveals how far we will go to protect our pride, but also because it directly hinders us from finding the kind of success that we idolize. When we decide that a goal is unattainable right from the start, we more easily justify our own inaction.

(3) Accepting defeat right from the start only ensures that we will never find success. In this sense, Gladwell’s theories are both supportive and discouraging. While Gladwell argues that immensely successful people do not possess some other-worldly abilities, he also posits that all types of factors need to line up for someone to achieve this level of success. In this way, his theory is encouraging because it means that many of us have the potential to achieve great success, but it is discouraging in that our success is still out of our control. In his book, Gladwell discusses Bill Gates as he was growing up. Yes, he was a very intelligent young man, but the level of his success is just as much a product of Gates growing up at a time when computer programming was just becoming possible, living in a place where he had access to a computer that could program, and all the other factors that went right for him. If a child is born right now with all the same inherent intelligence as Bill Gates and potential for computer programming, she isn’t likely to reach his level of success because he was there at the beginning. It’s like the difference between buying stock in Apple right now and buying stock in Apple 30 years ago. Bill Gates was learning programming before anyone realized how valuable it was, and that’s why he isn’t just a good programmer with a good job. What we sometimes forget, though, is that success is not black and white. That good programmer with a good job is successful even if she never reaches the kind of success that Bill Gates has achieved.

(4) Our natural inclination to put things into schemas and turn everything into dichotomous groups keeps us not only from finding success in our lives, but from understanding when we have achieved success. People don’t belong in groups where Bill Gates is successful and anyone who makes less money than him is unsuccessful. There are studies that show that happiness isn’t tied to this kind of success. In a well-publicized study, Angus Deaton and Daniel Kahneman posited that a person’s happiness increases as his income increases, but only until he earns about $75,000 a year; after that, returns begin to diminish. In other words, we can attribute success to income, but is there any indication that Bill Gates is happier than someone with a good job and a good income? Once we start realizing that we can be successful even if we aren’t rich and famous, we will live much happier lives. We may not all have the specific opportunities that someone like Bill Gates has, but we all have the ability to achieve success as long as we maintain a reasonable definition of the word.
 

Based on the passage, which of the following claims do you think the writer would agree with?

A
A person’s happiness has nothing to do with the amount of money they make.
B
Malcolm Gladwell’s book, Outliers, is a helpful guidebook for people trying to achieve success.
C
People tend to explain things in a way that makes them feel better about themselves.
D
A person really finds success when he or she starts a family.
Question 30 Explanation: 
The correct answer is answer choice (C). In the beginning of the passage, the writer discusses this idea when he discusses how we tend to think of incredibly successful beings as possessing “other-worldly” knowledge or skills. According to the writer, this makes our lack of success more palatable. Answer choice (A) is refuted by the writer’s discussion of the economic study in the final paragraph, and answer choice (B) is not an accurate representation of this writer’s analysis of Gladwell’s book. Answer choice (D) may be tempting because the writer encourages readers to define their own success, which for some would be having a family, but the passage suggests that the writer wouldn’t limit success to any one achievement.
Once you are finished, click the button below. Any items you have not completed will be marked incorrect. Get Results
There are 30 questions to complete.
List
Return
Shaded items are complete.
12345
678910
1112131415
1617181920
2122232425
2627282930
End
Return